Problema 2(N)

Versión para impresión
Sin votos (todavía)

Para un entero positivo n denotamos con S(n) la suma de los dígitos y con U(n) el dígito de las unidades. Determinar todos los enteros positivos n con la propiedad de que n=S(n)+U(n)2  (Nota: Para n=324, S(n)=9 y U(n)=4.)




Imagen de jmd

Gracias por la colaboración

Gracias por la colaboración Alain y felicidades por tu examen perfecto. 

Te saluda

Imagen de Roberto Alain Rivera Bravo

De nada y gracias, tan pronto

De nada y gracias, tan pronto tenga oportunidad subiré mis respuestas de cada problema. Saludos

Imagen de Weldersay

Alguna sugerencia para este

Alguna sugerencia para este problema por favor.

Por ahora tengo esto, $0\leq n-S(n)= U(n)^2\leq 81$ Ahora como 

$n$ y $S(n)$ dan el mismo resto $(mod9)$ entonces $n-S(n)= U(n)^2 \equiv 0(mod9)$

por lo tanto $U(n)$ es igual a uno de los valores $0,3,6$ ó $9$ luego trato de ver cada caso pero me queda por ejemplo.

$n-S(n)=3$  pero me queda dos incognitas... Debe de haber algo que no estoy viendo, hummm....

Imagen de jesus

Bueno, primero que nada te

Bueno, primero que nada te aclaro que comestiste un error en tu redacción, seguramente quisiste escrbir $n -S(n) = 3^2$ en lugar de $n-S(n) = 3$.

Sobre tu duda, sugiero que busques acotar por abajo el valor de $n - S(n)$ observa que si $n$ es muy grande $n-S(n)$ será muy grande. 

Observa que si $n -S(n) = 9$ debe ocurrir que $n$ tiene a lo más dos dígitos, o de lo contrario $n - S(n) \geq 99$.

Saludos,

Imagen de Weldersay

Si $n$ tuviera mas de tres

Si $n$ tuviera mas de tres digitos y el digito de las unidades fuera 9 entonces se tendria $n\geq109$ y $s(n)\leq27$ entonces $n-s(n)\geq82$ lo cual no es posible ya que $n-s(n)\leq81$ entonces nos queda $n-s(n)=U(n)^2\leq36$ en este caso similar al caso anterior si $n$ tuviera mas de tres digitos entonces $n\geq100$ y $s(n)\leq27$ por tanto $n-s(n)\leq73$ tampoco es posible, en definitiva  $n$  tiene como máximo dos cifras y su digito de las unidades es 0, 3 ó 6

el caso $n-s(n)=0$ es fácil de descartar

Si $n-s(n)=9$ ó $n-s(n)=36$ viendo los números $\{ 13,16,23,26,33,36,43,46,53,56,63,66,73,76,83,86,93,96 \}$ se puede ver que solo $13$  y  $46$ son solución.

PD. El error del post  anterior fue un mal tecleo.... jejeje.

Saludos.

Imagen de jesus

La desigualdad $s(n) \leq 27$

La desigualdad $s(n) \leq 27$ únicamente se vale cuando $n$ tiene a lo más 3 dígitos. Por lo que todo tu argumento para eliminar el caso en que $n$ tiene 3 o más dígito, sólo vale cuando $n$ tiene exactamente 3 dígitos. Pero no te debe tomar mucho corregir los argumentos para salvar esta demostración.

Por otro lado, para resolver el caso de dos dígitos tu argumento final es perfectamente aceptable. Sin embargo, yo preferiría usar la descomposición $n = 10a + b$ donde $a$ y $b$ son dígitos. Con esto, la identidad  $n - S(n) = U(n)^2$ se transforma en $9a = b^ 2$. Esto nos permite encontrar $a$ dado el valor de $b$, así que para los valores $b=0,3,6$ le corresponden los valores $a=0, 1, 4$; es decir las soluciones son 13 y 46 ( $a=0$ y $b=0$ se descarta por obvias razones).

En general, tu manejo de las desigualdades y organización del problema me parece correcto. Está todo bien escrito y salvo el detalle que mencioné al principio, la demostración es correcta.

Saludos

Imagen de Nayeli Aguilar

Te faltó 99 siendo

Te faltó 99 siendo (9+9)+9^2=99 18+81=99
Imagen de Weldersay

Ya que la prueba esta hecha

Ya que la prueba esta hecha para cuando n tiene menos de cuatro digitos entonces si tuviera cuatro o mas digitos tendriamos.
$n\geq 10^3$  y $S(n)\leq 9(4)$
$n\geq 10^4$  y $S(n)\leq 9(5)$
   .                               .
   .                               .
   .                               .
$n\geq 10^k$   y  $S(n)\leq 9(k+1) $ entonces $n-S(n)\leq 10^k-9(k+1)$ con $k$ entero positivo y $k\geq3$
Ahora probaremos que $10^k-9(k+1)>81$   para todo $k\geq3$
Por inducción.
Si $k=3$ entonces $10^3-9(4)=964>81$ que es cierto
supongamos que la proposicion es cierta para $k=n\geq3$,
$10^n-9(n+1)>81$
debemos demostrar que $10^{n+1}-9((n+1)+1)>81$
partimos de $10^n(10-1)>9$ con $n\geq3$
entonces $10^{n+1}-10^n>9 $ entonces $10^{n+1}-9>10^n$
luego $10^{n+1}-9-9(n+1)=10^{n+1}-9((n+1)+1)>10^n-9(n+1)>81$ lo que termina la prueba.
 
Que nivel tendría este problema?
PD. Presiento que tienes otra forma de descartar cuando n tiene tres o más digitos.
 
 Saludos.
Imagen de jesus

Con ese argumento queda

Con ese argumento queda completa la prueba.

La verdad sí tengo otra prueba pero es muy similar en complejidad y en el tipo de argumentos, una diferencia sustancial es que no uso inducción.

  • Si $n$ tiene $k$ dígitos se puede escribir como $$n = a_{k-1}10^{k-1} + a_{k-2}10^{k-2} + \dots a_2 10^ 2 + a_110^1 + a_0$$
  • Entonces, sustituyendo y luego aplicando la desigualdad ($a_i10^i - a_i \geq 0$ para todo $i$) se obtiene que  $$n -S(n) = \sum_{i=0}^{k-1} a_i10^i - a_i \geq a_{k-1}10^{k-1} -a_{k-1}$$
  • Pero si $k \geq 3$ tendremos que $k-1 \geq 2$, entonces tendremos que$$a_{k-1}10^{k-1} -a_{k-1} \geq a_{k-1}10^2 -  a_{k-1} = 99a_{k-1}  \geq 99$$ En La última desigualdad usa que $ a_{k-1} \geq 1 $ (ya que el dígito de más a la izquierda de $n$)
  • Entonces habremos demostrado que $n -S(n) \geq 99$ para todo $n$ de 3 o más dígitos.

Yo pondría este problema entre nivel avanzado o intermedio, no lo sabría. Pues bien cabría como un problema 1 o 4 de nacional. Este problema apareció en el selectivo del año pasado.

Saludos y gracias por compartirnos tu solución.

Imagen de German Puga

Hola, agregué la solución

Hola, agregué la solución ''oficial'' al problema problema. Estoy de acuerdo contigo Jesús, este problema tiene nivel medio o avanzado creo que es fácil de ver que n no puede tener más de dos digitos, pero como siempre ¿como se prueba algo tan sencillo? y bueno, las desigualdades vienen a hacer el truco. 

Creo que en el estatal el demostrar que $k \leq 3$ se podia hacer de manera informal, por que precisamente esas desigualdes hacen el problema avanzado.

Saludos

germán

Imagen de jesus

Qué cosas, ¿cómo se nos pasó

Qué cosas, ¿cómo se nos pasó la solución n=99?

En mi argumento dije, pues $b=0,3,6$ pero me faltó incluir a 9.

Es bueno saber que argumentos informáles son válidos en el estatal.

Saludos

Jesús

Imagen de Weldersay

Hola,  Creo que yo induje a

Hola, 

Creo que yo induje a cometer el error que yo habia cometido a Jesus,

En  el post 5 al hacer el análisis de n para tres dígitos y que el digito de las unidades fuera 9.Pues, cometí el error tambien de excluirlo para cuando n tuviera dos digitos, cosa que no podia ser, entonces al conjunto de números que se evaluarón también tenia que incluirse los que terminaban en dígito 9 o sea faltarón los números $\{19,29,39,49,59,69,79,89,99\}$

Las disculpas a Jesús, y  las gracias a Germán por hacerlo notar y compartir la solución.

Saludos.